Nothing Special   »   [go: up one dir, main page]

Question Bank MCQ: Prof. Ghada Elkhouly Professor of Psychiatry

Download as pptx, pdf, or txt
Download as pptx, pdf, or txt
You are on page 1of 301

QUESTION BANK MCQ

PROF. GHADA ELKHOULY


PROFESSOR OF PSYCHIATRY
QUESTION 1

 Which of the following fields of psychology attempts to understand and explain how
the thoughts, feelings, and behaviour of individuals are influenced by the actual,
imagined, or implied presence of others?
A. Clinical psychology
B. Cognitive psychology
C. Cultural psychology
D. Trait psychology
E. Biological psychology
QUESTION 2

 Which of the following sentences explain the term “phenotype" in a proper way?

A. The observable properties of the body and behavioural traits


B. The photographic representation of chromosomes in a single cell
C. The unique genetic makeup for every person
D. The typical representation of the genotype’s information
E. The person’s latent behavioural traits
QUESTION 3

 Which of the following statements is a the most characteristic of the Rapid Eye
Movement Sleep (REM)?

A. All muscles are paralysed


B. Occurs every 90 minutes
C. Dreaming is forgettable
D. No mental activity at all
E. EEG is slow with K complexes
QUESTION 4

 Which of the following concepts is the most closely related to “nature and nurture”?

A. Consciousness and unconsciousness


B. Id and ego
C. Operant and classical conditioning
D. Hereditary and environment
E. Continuity and discontinuity
QUESTION 5

 Visual information is carried to the brain by the optic nerve in the form of neural impulses.
Auditory information is also conveyed to the brain as neural signals.
 Which of the following is the most likely explanation for why we see visual information and
hear auditory information?

A. Unique quality of the message


B. Different sensory nerve pathways
C. Different sense organs
D. Different cortical perceptions
E. Different cortical regions receiving different neural messages
QUESTION 6

 Which of the following examples of learning could be explained by classical


conditioning?

A. A medical student who is learning how memory works


B. An executive who is afraid that she will lose her job
C. A child who, after a painful dental visit, has learned to fear the dentist
D. A dog that has learned to “sit up" for a food reward
E. A lady used to drink cinnamon before her menstruation
QUESTION 7

 Which of the following brain areas is widely considered to be principally important


in controlling memory?

A. Hippocampus
B. Thalamus
C. Medulla
D. Corpus Callosum
E. Pons
QUESTION 8

 Which of the following statements about IQ is true?

A. It is usually stable throughout life


B. It is rarely influenced by an individual’s culture
C. It is rarely influenced by an individual’s early learning
D. It is easily increased by formal training in specific subject areas
E. It is correlated with creativity
QUESTION 9

 Which of the following sentences is correct about “emotion”?

A. Emotion plays no role in an organisms’ survival & adaptation


B. Emotion is more important than behaviour and mind
C. Emotion is composed of three interacting physiological processes
D. Emotion has two system working together on facing a stressor
E. Emotions is predominantly unconsciousness
QUESTION 10

 Which of the following are the correct three successive phases of general adaptation
syndrome?

A. Reactive frustration, sympathetic arousal and parasympathetic inhibition


B. Attention, comprehension, and resistance
C. Alarm reaction, resistance, and exhaustion
D. Adrenal release, Cognitive appraisal, and Stomach ulceration
E. Comprehension, resistance, and exhaustion
QUESTION 11

 Which of the following is the most likely factor to affect our personality?

A. Skin colour
B. Facial features
C. Life experiences
D. Economic level
E. parents' job
QUESTION 12

 Which of the following statements could be reflect the concept of “confirmation


bias”?

A. Become autistic
B. Seek help from psychiatrist
C. Believe in astrology
D. Engages in risky behaviour
E. Decide to be vegetarian
QUESTION 13

 Ahmed forgot to pick his mother in law up at the airport.


 Which of the following approaches might suggest that forgetting was Ahmed’s
unconsciousness way of saying that he did not welcome her visit?

A. Humanistic
B. Cognitive
C. Psychodynamic
D. Behavioural
E. Developmental
QUESTION 14

 Zeinab is a 65 – year –old female has been healthy all her life until she experiences a
sudden grand mal seizure. A brain MRI scan reveals a poorly demarcated large mass with
central necrosis in the frontal lobe
 Which of the following is the most likely affected function by this mass?

A. Language
B. Vision
C. Smell
D. Posture
E. Concertation
QUESTION 15

 Gylan is a 43 – year – old female teacher comes to her primary care physician reporting that
her partner says that she sits up in bed and screams or cries out and appears extremely
frightened. She looks sleepy during these episodes. She does not recall their occurrence.
 Which of the following is the most likely affected sleep stage?

A. Rapid Eye Movement (REM)


B. Non rapid Eye Movement (NREM)
C. Non Rapid Eye Movements stage 1 and 2
D. Non Rapid Eye Movements stage 3 and 4
E. Non Rapid Eye Movements stage 1 and 4
QUESTION 16

 An infant acts relatively indifferent to her mother’s presence, does not seem greatly
disturbed by her departure, and does not go to her when she return.
 Which of the following types of attachment styles is this infant?

A. Secure
B. Disordered
C. Avoidant
D. Preoperational
E. Ambivalent
QUESTION 17

 People usually respond not just to situations


 Which of the following is the most likely factor determine the difference between
different people’ responses?

A. Intelligence
B. Perception
C. Traits
D. Needs
E. Motives
QUESTION 18

 Tia is a girl child 6-years-old, when wanted a toy from her baby sister, she often grabbed it. Tia’s mother
did not allow her to keep the toy because that rewarded her for grabbing toys. However, Tia persisted. To
help his daughter change, Mr. Omar developed another plan; he gives her time out when she grabbed a toy.
 Which of the following is the used operant techniques by Tia’s mother and father respectively “in order”?

A. Negative punishment / positive punishment


B. Positive punishment / negative punishment
C. Negative reinforcement / positive punishment
D. Positive punishment / negative reinforcement
E. Negative punishment / positive reinforcement
QUESTION 19

 After an airplane crash in March 2015, actor Harrison Ford sustained serious injuries that hospitalized him
for nearly a month, and he also suffered gaps in his memory that left him unable to recall details of the
accident. Ford was piloting a World War II vintage aircraft when the engine failed. Nearby air traffic
controllers suggested a landing procedure for the ailing plane, and that was the last thing Ford remembered
until he woke up in the hospital five days later.
 Which of the following is the most likely type of memory loss ford suffered from?

A. Anterograde amnesia
B. Retrograde amnesia
C. Psychogenic amnesia
D. Traumatic amnesia
E. Organic amnesia
QUESTION 20

 Samar is a patient who moves frequently because she says her neighbors are conspiring to rob
her, and who is unable to keep a steady job because she always comes to believe that her co-
workers are plotting against her
 Which of the following is the most likely affected mind function?

A. Behaviour
B. Intelligence
C. Thinking
D. Memory
E. Mood
QUESTION 21

 Yasmine is a 3-year-old girl is evaluated in the pediatric general practice clinic because her mother think she
is “masturbating”. She notes that, she used to do frequent friction of his genitalia against a woody surface. In
addition, she worries that she sits doing this for hours rocking back and forth till becoming sweaty, with
flushed face and hyperventilation. She believed that she reaches orgasm. Her father thinks that, she exposed
to sexual harassment and start to fight with the mother and accused her for being neglecting her child.
 Which of the following is the most likely proper advice must given to Yasmeen’s family?

A. Recommend circumcision in the near future to deal with Yasmine’s hyper sexuality
B. Punish Yasmine every time doing this behaviour
C. Search in family and school about source of sexual harassment
D. Neglect Yasmine as well as her behaviour
E. Let Yasmeen paly gymnastics to deal with her high sensation seeking in a positive way
QUESTION 22

 Maram is a 27-year-old woman, married 5 years and the mother of 2 children, comes for an initial outpatient
appointment relating a history of “anxiety attacks”, characterized by a racing heart, nausea, shortness of breath,
trembling, and fear of dying. The attacks began about 2 months ago, following a minor traffic accident in which she
was involved. Since then, she has had 2 to 3 attacks / week. The attacks usually occur at the shopping market, when
going out to dinner, or when driving. Although she continues to go out, she now hates the thought of having to drive
and avoids it if at all possible.
 Which of the following is the most likely reason of Maram's poor stress management?

A. Low cognitive hardiness


B. Resilience mind set
C. High commitment
D. Proper control
E. Adequate challenge
QUESTION 23

 Rasheed is friendly, always willing to help others and compassionate.


 Which of the following personality traits, you would expect Rasheed to score highly on
it?

A. Extraversion
B. Agreeableness
C. Neuroticism
D. Openness to experience
E. Conscientiousness
QUESTION 24

 Mayar is a 17-year-old male visits his primary care provider because his parents are worried about his behavior. They
complain that he is always in the bathroom, taking hours during showering in the morning and frequently spending long
periods of time in hand washing throughout the day. He explains that, he has to make sure to get rid of all the germs on his
body, and that he has to make sure they are all gone before he leaves. His hands are somewhat dry and chapped. The
primary care provider writes a referral form to a psychiatrist as he thinks of obsessive compulsive disorder as a
provisional diagnosis. The parents refused this referral and said that, our son is a religious wised intelligent person; not in
need for a doctor who deal with mad people.
 Which of the following is responsible for his parents' response?

A. Social role
B. Socioeconomic class
C. Social norms
D. Personality characters
E. Religious commitment
QUESTION 25

 Who of the following individuals is credited with discovering the connection between
mind and brain?

A. B. F. Skinner
B. Phineas Gage
C. Ivan Pavlov
D. J. D. Watson
E. H.M
QUESTION 26

 A patient with a stroke complained of loss of ability to speak in a fully grammatical


sentences.
 Which of the following cerebral lobes is probably damaged?

A. Frontal
B. Parietal
C. Temporal
D. Occipital
E. Occipito-parietal
QUESTION 27

 Which of the following questions is the most likely informative about your sleep
deficiency?

A. Do you rarely fall asleep while reading or watching TV?


B. Do you awake in the morning feeling that you are rested?
C. Do you sleep early on weekends?
D. Would you oversleep if you use an alarm clock to drive you out of bed?
E. Do you often get sleepy in your classes?
QUESTION 28

 Which of the following types of development belonged to Jean Piaget’s theory?

A. Temperament
B. Cognitive
C. Psychosocial
D. Psychomotor
E. Psychosexual
QUESTION 29

 Which of the following is known by lay people as the “third eye”?

A. Perception
B. Imagination
C. Intelligence
D. Mind
E. Intuition
QUESTION 30

 Which of the following is a statement with which Skinner’s (founder of operant


conditioning) followers would agree?

A. Nature is more influential than nurture


B. Most behaviour is controlled by unconscious forces
C. The goal of behaviour is self actualization
D. Human is an active voluntary learner
E. Observation is the most important learning tool
QUESTION 31

 We organize much of our declarative memory from general to specific


 Which of the following is the most likely form for this storage?

A. Scattered concepts
B. Concept hierarchies
C. Raw data
D. Mental prototypes
E. Mental images
QUESTION 32

 Which of the following is best described as a measure of the intelligence quotient


(IQ)?

A. Innate cognitive development


B. Future cognitive development
C. Environmentally determined cognitive skill
D. Present functional cognitive ability
E. Learned verbal skills
QUESTION 33

 Which of the following sentences goes with Abraham Maslow’s hierarchy of needs?

A. All organisms strike to be as human like as possible


B. Basic lower needs must be satisfied before higher order needs
C. A need for esteem and self worth are more than other needs
D. Sexual desire is the hidden motive behind nearly all behaviours
E. Motivation has no relation to our needs
QUESTION 34

 Which of the following is a good advice for coping with student’s stress?

A. Take care of social duties rather than academic obligations


B. Rely on problem focused coping “take action to fix what is wrong”
C. Resist the urge to ask for help
D. Rely on emotion focused coping “make person feel better but do not help”
E. Vent your feelings frequently and intensely
QUESTION 35

 Which of the following statements was most closely associated with the discovery
that, the heritability of the big five personality traits is around 40-50%?

A. The environment plays no role in personality


B. Genes plays no role in personality
C. The environment plays an important role in personality
D. The big give traits account for about half of our personality
E. The big five traits transmit from parents to half of their children
QUESTION 36

 If you believe that most vegetarians wear sandals, ride bicycles, and support liberal
social causes.
 Which of the following is the most likely bias you used to judge vegetarians?
A. Confirmation
B. Anchoring
C. Availability
D. Representativeness
E. Hindsight
QUESTION 37

 Ghada is 28 - year-old woman comes to the emergency room speaking rapidly and reporting that she has not
been sleeping for the last week but feels very energetic. She states that she believes she has learned the
ultimate secret to ensure making a killing on the stock market. She has a history of appendectomy 5 years
ago.
 Which of the following specialities you prefer to call for consultation?

A. Psychologist
B. General Psychiatrist
C. Consultation Liaison Psychiatrist
D. Neurologist
E. Clinical Psychologist
QUESTION 38

 Ramy is driving on a winding mountain road, and suddenly a car is coming directly at him. At the last
instant, he and the other driver swerve in opposite directions. His heart bounds – and it keeps pounding for
several minutes after the danger has passed. Externally, he has avoided a potentially fatal accident.
Internally, his body has responded to two kinds of messages from its two communicating system.
 Which of the following systems were responsible for these two kind of messages?

A. Rapid ssympathetic and slow parasympathetic


B. Slow nervous and the fast endocrine
C. Fast nervous system and slow cardiovascular
D. Fast nervous system and slow endocrine
E. Slow nervous and fast cardiovascular
QUESTION 39

 Hamdy is a middle age man, obese came to your clinic with his wife, complaining of fatigue,
daytime sleepiness, & laziness. His wife told you that, his sleep is characterized by several brief
interruptions when he stops breathing, wakens, resumes breathing, and falls asleep again.
 Which of the following is the most likely diagnosis of hamdy’s case?

A. Insomnia
B. Analgesia
C. Sleep apnoea
D. Daytime sleepiness
E. Depression
QUESTION 40

 Waleed is a diabetic child came to your clinic with his parents. His blood sugar is uncontrolled and his
parents complaining of his non compliance. The child looks depressed, tense and refuse to talk in
front of his parents. On talking to him privately, he complains that his parents criticize him all the
time, order him to take his insulin in front of anyone.
 Which of the following parenting styles is the most suitable one to improve Waleed’s compliance?

A. Permissive
B. Uninvolved
C. Authoritarian
D. Authoritative
E. Overprotective
QUESTION 41

 Just prior to her seizures, Hala has a strange sensation (an aura) that feels like
pinpricks on her left leg.
 Which of the following brain lobes is the most likely responsible for these sensation?

A. Frontal
B. Parietal
C. Temporal
D. Occipital
E. Insula
QUESTION 42

 Nawal is about to undergo her first round of chemotherapy. To her surprise, the nurse enters
the lab, not with the expected syringe, but with a dish of licorice flavoured ice cream. Is this a
new kind of therapy? She asks. The nurse replies that it is, indeed.
 Which of the following is most likely adopted learning theory by this therapy?

A. Operant conditioning
B. Classical conditioning
C. Cognitive
D. Observational
E. Social
QUESTION 43

 Mostafa is a patient exposed to a blow to the head. He experienced loss of recent memories.
 Which of the following is the most affected stage of memory could lead to his amnesia?

A. Consolidation
B. Storage
C. Retrieval
D. Understanding
E. Perception
QUESTION 44

 Ragheb is a 26 years old man believes that the Mafia & his brother are putting
horrible thoughts into his head
 Which of the following Regheb’s mind functions is affected?

A. Thoughts
B. Mood
C. Memory
D. Behaviour
E. Personality
QUESTION 45

 Omar is a boy 5 years old. He has the following proposal; if he will wait until after his father runs an
errand, he can has 2 marshmallows for a treat. If he can not wait then, he can only have 1 – but his
father can have it right now. He choose to wait and take the 2 marshmallows.
 Which of the following is the most likely type of intelligence Omar used to postpone his gratification?

A. Musical
B. Academic
C. Social
D. Practical
E. Emotional
QUESTION 46

 Ramzy is a 65 year-old man admitted to your hospital. During the round, he asked
you to feel more control on his daily activities like meals & TV time. Which of the
following is the reason for your agreement for his ask?

A. Prevent learned helplessness


B. Prevent negative transference
C. Encourage Cognitive hardiness
D. Build good doctor patient relationship
QUESTION 47

 Yassin is a friend of yours always seems agitated & anxious, even when nothing in the
circumstances would provoke such a response
 Which of the following fig five traits is the most characteristic one of Yassin?

A. Neuroticism
B. Introversion
C. Extroversion
D. Agreeableness
E. Conscientiousness
QUESTION 48

 In the hospital, each group of health team as well as patients, must put uniform
specific for each career
 Which of the following is the most likely name for this rigid dress code?

A. Social norms
B. Health golden rule
C. Identity code
D. Marketing law
E. Strict regimen
QUESTION 49

 From a medial point of view, Which of the following components form the mind
(Psyche / Nafs)?

A. Spirit, soul, and body


B. Heart, brain, and soul
C. Thoughts, mood(s), and behaviour(s)
D. Thought, intelligence and mood
E. Consciousness, pre-consciousness, & unconsciousness
QUESTION 50

 Which of the following statements about brain is true?

A. The brain senses the world indirectly as the sense organs convert stimulation into
neural messages
B. The brain senses the world indirectly as the motor system convert stimulation into
neural messages
C. The brain senses the world directly as the motor system convert stimulation into neural
messages
D. The brain senses the world directly as the sense organs convert stimulation into neural
messages
QUESTION 51

 Which of the following is one of the functions of the consciousness?

A. Distracting attention from what is relevant


B. Manipulating a mental image of the world
C. Relinquishing control to enhance self awareness
D. Dividing sensation and memory
E. Controlling night dreams
QUESTION 52

 Which of the following best exemplifies a developmental change that results from an
interaction of hereditary & environment
A. Eye colour
B. Learning to talk
C. Winning a lottery
D. Facial hair in adolescent
E. Skin colour
QUESTION 53

 Which of the following is the most likely function perception adds to sensation?

A. Meaning
B. Intensity
C. Positivity
D. Energy
E. Depth
QUESTION 54

 Insight Learning, observational learning and cognitive maps all are belonged to one type of
learning
 Which of the following is the most likely type of learning described all of those examples?

A. Cognitive
B. Behavioural
C. Simple
D. Technical
E. Instrumental
QUESTION 55

 Which of the following is the memory system known to reconstruct material during
retrieval?
A. Human
B. Computer
C. Animal
D. Mobile
E. Video
QUESTION 56

 Which of the following could represent a concept hierarchy?

A. Woman, girl, man, boy


B. Lemur, monkey, chimpanzee, human
C. Animal, mammal, dog, cocker
D. Cat, dog, giraffe, elephant
QUESTION 57

 Which of the following is the most accepted from research as being the ability to
control one’s emotional responses?

A. Personality trait
B. Hormonal profile
C. Intelligence level
D. Learned skill
E. Inherited genotype
QUESTION 58

 Which of the following is an example of individualistic stressor?

A. Sexual rape
B. Earthquake
C. Military combat
D. Birth of first child
E. Loss of saved money
QUESTION 59

 You want to understand your patient’s personality. Which of the following is the least
helpful factor for you?

A. Situation
B. Age
C. Traits
D. Socioeconomic class
E. Motives
QUESTION 60

 Which of the following would be an example of the confirmation bias?

A. Yehia archive his close friends on WhatsApp


B. Yehia agrees with Hassan’s taste in music
C. Yehia buys a car, even though his wife was opposed to the purchase
D. Yehia ignore negative information about her favorite singer
E. Yehia refuses to eat a food she dislikes
QUESTION 61

 On a psychiatric examination, which of the following is a clue to suggest diagnosis of


schizophrenia (though disorder) rather than a depression (mood disorder)?

A. The patient believes that police want to catch him


B. The patient is poorly adapted to his daily activities
C. The patient feels bad about ordinary life events
D. The patient has sleep onset insomnia
QUESTION 62

 A 62 – year – old married man was admitted to a surgical ward following an acute abdomen. Three days
after an exploratory surgery, he complained of feeling anxious, shaky and mildly confused. He is
experiencing vivid visual hallucinations.
 Electroencephalogram (EEG) showed spike and slow wave complexes
 Which of the following is the most likely brain area affected in the patient?

A. Frontal lobe
B. Temporal lobe
C. Parietal lobe
D. Insula
E. Amygdala
QUESTION 63

 If you are working in a sleep laboratory, where you are monitoring a subject’s sleep
recording during the night.
 At the night progress, which of the following would you suspect to see?

A. The four stages cycle gradually lengthens


B. REM period becomes longer
C. Stages 3 & 4 NREM sleep periods lengthens
D. Dreaming become less frequent
E. Snoring will must occur during deep sleep
QUESTION 64

 Salam is a 2-year-old boy presented to your clinic because his mother thinks he is “too quiet”. She noted
specifically that her pregnancy and delivery were uncomplicated. A few months after birth she noted that her
child did not seem to respond to her attempts to communicate with him probably. He doesn’t play with other
children and overall shows little interest in his external world. She is so concerned about her child and asked
you for a precious advice in order to help him.
 Which of the following is the most helpful advice you can give her in order to help her child?

A. Excessive sleep is good for him


B. Pharmacological stimulation of the brain is mandatory
C. Postpone school as late as possible
D. Heavy sensory stimulation of the brain is mandatory
E. Set restrict rules about his daily activities
QUESTION 65

 A 28-year-old woman tells her gynecologist that she has never had a complete sexual intercourse since marriage.
She says that when they are together she refused vaginal penetration. She has active desire, but whenever, her
husband attempts to enter her vagina she feel severe pain and her muscles contracts. The doctor said; this is a
known case of vaginismus. She started a program for managing pain through anatomical explanation, build
couple trust, counseling with husband, anesthetic spray, and using competing pain during the sexual play.
 Which of the following is the most applied theory on her management plan?

A. Placebo
B. Gate control
C. Transduction
D. Gestalt
E. Psychodynamic
QUESTION 66

 Tamer, following a minor fight in school, develops weakness of his right leg. No physical
cause can be shown for the leg weakness. However, his leg’s weakness becomes so marked
that he cannot go to school
 According to learning theories, which of the following is the most likely explanation for
intensification Tamer's leg weakness?

A. Positive reinforcement
B. Positive punishment
C. Negative reinforcement
D. Negative punishment
QUESTION 67

 Karam is an unemployed, college – educated 36 –year-old who has a well-documented history of


sexual and physical abuse by multiple male family members. At least three persons have been
identified. He has intrusive thoughts, autonomic over arousal, nightmares, and a history of self-harm.
 Which of the following types of memory is the place of storing these painful personal information?

A. Implicit
B. Procedural
C. Semantic
D. Working
E. Episodic
QUESTION 68

 Hazem is a young child was asked why the sun sets? Immediately the answer came
“because Hazem must go to sleep”.
 Which of the following is the correct cognitive stage of Hazem?

A. Animism
B. Accommodation
C. Assimilation
D. Egocentrism
E. Theory of mind
QUESTION 69

 When a person is becoming angry, if he starts observing and studying his sate of mind
simultaneously, he will not be able to show his anger. The moment he starts observing his own anger,
it may subside. This problem can be partially solved by observing the experience after is over.
 Which of the following is the most likely feature of this person?

A. Emotional intelligent
B. Social intelligent
C. Type A personality
D. Internal locus of control
E. High cognitive hardiness
QUESTION 70

 A 23-old-woman who was raped 5 months ago complains of recurrent thoughts of that event
every time any one touches her. She states this has been happening for the past 2 months often
accompanied by nightmares that wake her up at night.
 Which of the following is the type of her stressor?

A. Generalized
B. Individualistic
C. Ordinary
D. Moral
E. Cultural
QUESTION 71

 Kamal who experiences extreme anxiety undergoing cardiac bypass surgery is referred to
a psychiatrist for evaluation.
 Which of the following is the most likely contributing factors for the Kamal's symptoms?

A. Personality Character
B. Birth of order
C. Socioeconomic class
D. Age and sex
E. Body weight
QUESTION 72

 A 25-year-old woman presents with a 2 week history of being unable to walk, which began suddenly
after fighting with her husband. Her husband divorced her verbally. On examination, she is able to
maintain tone in her legs, but is unable to walk. Her mother convinced that, she is controlled by Gene
and ask for the help form traditional healer.
 Which of the following is the context by which we can understand the mother’s conviction?

A. Cultural
B. Religious
C. Health
D. Cognitive
E. Legal
QUESTION 73

 Which of the following is the most likely definition of psychology?

A. The science of behaviour and mental processes


B. The science of personality and mental illnesses
C. The science of the mind and brain
D. The science of behaviour and biology
E. The science of personal growth and self fulfilment
QUESTION 74

 Which of the following neurotransmitters that regulates sleep, dreaming, mood,


pain, aggression, appetite & sexual behaviour?

A. Dopamine
B. Noradrenaline
C. Serotonin
D. Glutamate
E. Acetyl Choline
QUESTION 75

 Which of the following is the most likely description of the consciousness?

A. Consciousness interfere with unconsciousness


B. Consciousness controls the ANS
C. Consciousness allows us to respond reflexively, without thinking
D. Consciousness process information serially
E. Consciousness makes us more alert
QUESTION 76

 Which of the following statements described the influence of parents on the


personality of their children?

A. Non existent
B. Weakest in early childhood
C. Consistent across life span
D. Strongest in early childhood
E. Most importantly at adolescence
QUESTION 77

 Not all stimuli become sensations.


 Which of the following is the most likely explanation for this sentence?

A. Some stimuli are subthreshold


B. Some stimuli are out of our interest
C. Some stimuli are changeable
D. Some stimuli are contradictory
E. Some stimuli are changeable
QUESTION 78

 Which of the following is the distinguishing feature characterize operant


conditioning?

A. Events such as rewards & punishments occur before the behaviour


B. Events such as rewards & punishments occur after the behaviour
C. Events such as rewards & punishments occur during the behaviour
D. Events such as rewards & punishments occur with the behaviour
QUESTION 79

 During a football game, a player sustains a powerful blow to the lateral side of his head. He
experienced anterograde amnesia. His coach start to train him by techniques known to improve
memory. For example; he associates names of other players in a sequence correspond to the player’s
Nike name.
 Which of the following is the mnemonic used by the coach?

A. Natural language mediators


B. Absent mindedness
C. Method of loci
D. Expectancy bias
E. Misinformation effect
QUESTION 80

 Which of the following is a heuristic technique?

A. Optimism
B. Fixation
C. Similarities
D. Determination
E. Summation
QUESTION 81

 Which of the following is the final destination for much of the brain’s information
about emotion before action is taken?

A. Amygdale
B. Anterior cingulated cortex
C. Pre-frontal cortex
D. Hypothalamus
E. Medulla
QUESTION 82

 Which of the following is the first physical response to stress?

A. Fear & irritability


B. A state of arousal
C. Reduction in immunity
D. A protective behaviour
QUESTION 83

 Which of the following statements is a proper definition of “personality traits”?

A. Physical characteristics that distinguish us from other people


B. Relatively enduring characteristics that influence our behaviour across many
situations
C. Unconsciousness tendencies to act in different ways according to the situation
D. Permanent personality tendencies that determine our behaviours in any situation
E. Changeable characteristics according to learning and experience
QUESTION 84

 Prejudiced people usually avoid information that conflicts with their view of the
world, so they never watched or listened to information campaigns & education
 Which of the following is the most likely name of this attitude?

A. Confirmation bias
B. Scapegoating
C. Discrimination
D. Stereotypy
QUESTION 85

 Faten is a 39 year old woman comes to the emergency department of a general hospital with her
husband. She is agitated, diaphoretic, and tremulous. She cries out in fear of visual hallucinations.
She has a history of one myocardial infarction and 20 years of insulin dependent diabetes mellitus.
 Which of the following is the most likely statement for proper management of her crying?

A. It is irreverent to ask about the cause of her crying, she is just tired
B. It is a must to postpone the asking about her crying to follow up
C. It is a must to ask her directly about the cause of her crying
D. It is a must to ask her husband about cause of her crying
E. It is a must to document the crying in her clinical history without asking
QUESTION 86

 Othman is my friend’s dad. He is a clever lawyer. Last week he behave in a very strange manner
while defending a case in the court. He stand over the bench and urinate in front of all the audiences.
 Which of the following brain lobes is most likely affected by vascular insult as revealed from MRI?

A. Frontal
B. Occipital
C. Parietal
D. Temporal
E. Tempo-parietal
QUESTION 87

 Sohair is a girl 21 years old, come to your clinic with her sister who will pass a liver transplantation.
The patient suggest her sister as a donner; suppose you want to sample the contents of her
unconsciousness in order to verify her acceptance for donation; which technique would be most
appropriate
 Which of the following is the most likely method to assess her verification?

A. Asking her a direct question


B. Zooming in on mental image
C. Using a prime technique
D. Doing mental rotation experiment
E. Asking her intimate friend
QUESTION 88

 Mahmoud is afraid to be so impulsive and moody like his uncle.


 Which of the following is the most likely acceptable scientific sentence to reassure Mahmoud with?

A. Heredity is stronger than environment


B. Hereditary acts alone
C. You are not related genetically to your uncle
D. Both hereditary & environment work together to influence our behaviour & mental processes
E. Hereditary determines our psychological destiny
QUESTION 89

 Nana is a 65 year old passed an automobile accident damaged the brain’s region processed colour
information. She lost colour vision, memory of colour and eventually the names of colours. She
could no longer even imagine, for instance, what red once looked like
 Which of the following statement is the most likely demonstration of her experience?

A. Colours do really exist out there


B. The world of colour constructed by the sensory and perceptual process outside the brain
C. We don’t experience the world directly, but instead through a series of filters that we call our senses
D. The red colour has specific characters rather than other colours
E. Trauma is so stressful that interfere with her quality of life
QUESTION 90

 Children jump when they hear an unexpected loud noise. This behaviour dose not produce
any lasting change in the brain. It is nothing more than a fleeting reaction, even thought it
does not entail a change in behaviour
 Which of the following is the most likely description of the behaviour?

A. Learned
B. Reflexive
C. Instinctual
D. Thoughtful
E. Consciousness
QUESTION 91

 Zafer is a patient lost most of his ability to form new declarative memories as a result of a botched
brain operation. Since then, he has been unable to create new memories of the events in his life. New
experiences slip away before he can store them in long term memory – although his memory for
events prior to the operation remains normal.
 Which of the following types of memory you expect to be intact after his operation?

A. Episodic
B. Declarative
C. Procedural
D. Emotional
E. Photographical
QUESTION 92

 A bus along the freeway suddenly begins sputtering and then quits. As the driver
coast over to the shoulder, he noticed that the gas gauge says empty.
 Which of the following is the most likely action the driver must do?

A. Check the battery cable


B. Phone your machinist for advice & help
C. Hike to the nearest service station for a gallon of gas
D. Consider all the possibilities before committing to one solution
QUESTION 93

 Maha is an obese patient. She is 25 years old. She has stress and depression affecting her appetite. She
associates certain situations with eating, so she feel hungry regarding of her biological needs. Her dietician
refer her to psychiatrist to deal with her problem.
 Which of the following is the most likely eating problem psychiatrist will deal with?

A. Excessive
B. Emotional
C. Unbalanced
D. Poor
E. Biological
QUESTION 94

 Hanan decides to be feel more happier this year


 Which of the following is one of the things she might do to achieve her goal?

A. Earn a much higher income


B. Develop a more involved spiritual dimension in her life
C. Spend more time on her own & less involved with other people
D. Evaluate the world around her more realistically even if it is less positive
QUESTION 95

 On the day you must prepare for a major test, a friend confides in you about a terrible
problem and begs for your help. These 2 stressors – an important test & needy friend -
could be overwhelming, especially if you feel you are already stretching some of your
resources to the limit.
 Which of the following attitudes will be most helpful for you?

A. challenge, effectiveness & patience


B. commitment, obedience & victimization
C. control, efficiency & seriousness
D. challenge, commitment & control
QUESTION 96

 Interviewing medical patient includes asking about his/her original & current address.
 Which of the following is the main reason for concerning these questions in the
management plan?

A. Culture
B. Biology
C. Genetics
D. Stress
E. Career
QUESTION 97

 A little girl Lina pushed Omar, her brother, off his tricycle. She learned to behave this way
because the behaviour paid off in the past, in other words, she learnt to act aggressively in
certain situations because she was rewarded for such behaviour in the past.
 Which of the following perspectives is the most suitable to study Lina’s problem?
A. Cognitive
B. Social
C. Biological
D. Developmental
E. Behavioural
QUESTION 98

 Which of the following is the brain’s ability to adapt or modify itself as the result of
experience?

A. Plasticity
B. Action potential
C. Transduction
D. Priming
E. Cognitive hardiness
QUESTION 99

 Consciousness changes in cycles that normally corresponds to our biological rhythms


and to the patterns of our environment
 Which of the following is a proper illustration for this concept?

A. REM period
B. Priming
C. Sleep & dreaming
D. consciousness, preconscious & unconsciousness
QUESTION 100

 Which of the following will show decline by old age?

A. Vision and hearing


B. Emotional wellbeing
C. Social support from family
D. Thinking and problem solving abilities
E. Procedural memory
QUESTION 101

 Maya is 19 years old. She is a student in the first year of the faculty of medicine. She
complained of inability to concentrate during study. Her father advice her to replace
interesting music by bored one while studying.
 Which of the following concepts which reason the advice if Maya’s father?

A. Sensory adaptation
B. Unlimited unconsciousness
C. Stimulus threshold
D. Transduction
QUESTION 102

 We take aspirin to relief pain


 Which of the following is the most likely explanation for this relief?

A. Positive punishment
B. Positive reinforcement
C. Negative punishment
D. Negative reinforcement
QUESTION 103

 Which of the following are the 3 basic tasks of memory?

A. Sensory, working, & long term


B. Encoding, Storage & retrieval
C. Recall, recognition & relearning
D. remembering, forgetting & repressing
E. Encoding, understanding & storage
QUESTION 104

 Which of the following is a component of thought?

A. Stimuli
B. Images
C. Impulses
D. Tics
QUESTION 105

 Motivation takes many forms, but all involves inferred mental processes that select
& direct our behaviour.
 Thus, the psychology of motivation attempts to explain why a certain ------- is
selected?

A. Emotion
B. Reward
C. Sensation
D. Action
QUESTION 106

 The human stress response to perceived threat activates several psychological


reactions
 Which of the following is the main aim of these reactions?

A. Threaten health & survival


B. Slow & reduce one’s ability to respond
C. Produce increasing pain
D. Promote adaptation & survival
QUESTION 107

 Which of the following is characteristic to temperament theory of personality in


contrast to other theories like psychodynamic, humanistic, & cognitive ones?

A. A description of the components of personality


B. A description of the processes of development & change underlying personality
C. Labels for common mental disorders
D. Concepts that are useful for individuals involved in personnel selection decisions
QUESTION 108

 Which of the following is the most likely strategy for getting people liking you?

A. Tell them the truth


B. Hide your interests from them
C. Keep your feeling towards hem as secret
D. Persuade them to perform a difficult or unpleasant task for you
QUESTION 109

 Manal is a 15 – year – old girl’s menstrual periods have stopped, and her weight has dropped from 70 Kg to
44 Kg during the last 3 months. No other symptoms are reported, and the girl is unwilling to discuss any
problems with her physician. Physical examination and routine laboratory tests reveal no abnormalities. The
girl has a good school attendance record and average grades; other behavioural information has not been
requested
 Which of the following professions is highly needed in the Manal's management medical team?

A. Clinical Psychologist
B. Neurologist
C. General Psychiatrist
D. Social worker
E. Consultation Liaison Psychiatrist
QUESTION 110

 Which of the following substances is a neurotransmitter?

A. Insulin
B. Dopamine
C. Tyrosine
D. Thyroxine
E. Nitrogen
QUESTION 111

 Basem awaken some nights gasping for air, and his roommate tells you that he snores
loudly, even when he is just dozing.
 Which of the following is consistent with Basem’s symptoms?

A. Sleep apnoea
B. Narcolepsy
C. Insomnia
D. Chronic fatigue
E. Confusion
QUESTION 112

 A 12 – year – old boy is brought to the outpatient clinic by his parents as they are very concerned about
him. He has always been a loner. He has been having problems at school recently. He is being bullied
very often as he tends to do solitary activities at school.
 On examination, there is no evidence of learning disability or speech delay.
 Which of the following is the most likely deficient developmental need in this boy?

A. Socialization
B. Attachment
C. Mental schema
D. Stimulation
E. Contact comfort
QUESTION 113

 A 9 year old boy has his headphones on. A random number of digits are being spoken in his
right ear and a random number of words are spoken in his left wear at the same time. The
task for him is to repeat what he hears in his left ear.
 Which of the following is the most likely helpful function for this boy?

A. Perception
B. Memory
C. Attention
D. Intelligence
E. Thinking
QUESTION 114

 Mostafa is a 15 year old male is brought to his paediatrician’s office because his parents are
embarrassed by his behaviour. They report that he has been intermittent barking and yelling
repetitively during conversations. This behaviour persists in public and at school. His
parents believe he is punishing them for setting limits on TV & social time with his friends.
 Which of the following is the used operant conditioning technique by parents?

A. Negative reinforcement
B. Negative punishment
C. Positive reinforcement
D. Positive punishment
QUESTION 115

 Hala had written a list to do her week and work, but incidentally left it home while going
out for her first activity. Trying to remember the list, Hala remembers what was at the
beginning of the list and what was at the end but not those things in the middle.
 Which of the following is the most likely affected type of memory?

A. Explicit
B. Implicit
C. Procedural
D. Sematic
E. Flash bulb
QUESTION 116

 you: you are so fragile


 Your friend: I can not changed
 You: why? Of course you can
 Your friend: no I can not, this is a trait, I looks like my mother
 Which of the following is supportive for your opinion?

A. Our personality is a genotype


B. Our brain is a computer metaphor
C. Nerve cells stop to grow after puberty
D. Observational learning is very strong in development
E. Change occurs only after stress
QUESTION 117

 Huda just does not enjoy doing much of anything lately. She blames herself for all of her
failures in life. She has lost interest in food and social activities, and most days she stays in
bed.
 Which of the following is the most likely basic emotions colour Huda's case?

A. Fear
B. Sadness
C. Anger
D. Contempt
E. Joy
QUESTION 118

 Jody is a 24 – year – old woman is being evaluated for depression and anxiety. She reports
that she has been stressed about her husband because he “lost his business” and her parents
“have always been really disapproving of him”.
 Which of the following is the type of Judy's stressor?

A. Individual
B. Generalized
C. Traumatic
D. Emotional
E. Threatening
QUESTION 119

 A patient who, when diagnosed with cancer, refused the diagnosis of malignancy and
does not comply with the recommended treatment
 Which of the following is the most likely defensive mechanism used by this patient?

A. Projection
B. Sublimation
C. Denial
D. Reaction formation
E. Repression
QUESTION 120

 World health organization defines the female genital mutilation (FGM) as a crime. In FGM a
vital sex organ is excised. This leads to disturbed sexual satisfaction in marital life.
 Which of the following will be the main obstacle to highlight this information in the general
population?

A. Culture
B. Religion
C. History
D. Modelling
E. Health
QUESTION 121

 Which of the following is the most likely order for assessment of any psychiatric
patient/client?

A. Thoughts / mood / behaviour


B. Mood / behaviour / thoughts
C. Behaviour / thoughts / mood
D. Behaviour / mood / thoughts
QUESTION 122

 Which of the following might carry a neural impulse across the synapse?

A. Cerebrospinal fluid
B. Dopamine
C. Plasma
D. Electrical impulse
E. Hormone
QUESTION 123

 Which of the following is the most likely is a description of both


consciousness/unconsciousness?

A. Both are reactive


B. Both are present oriented
C. Both are voluntary
D. Both are restricted
E. Both are cooperative
QUESTION 124

 Which of the following is the major demographic shift is now in progress?

A. A culture that is increasingly focusing on youth


B. An increase in the average age of the population “gerontology”
C. The roles of the worker and parent becoming more rigidly defined
D. Fewer women assuming professional roles
E. Birth control for all couples more than 30 years old
QUESTION 125

 Which of the following is the reflecting idea of Gestalt’s “law of closure”?

A. Continuity
B. Completion
C. Contouring
D. Commonality
E. Similarity
QUESTION 126

 A bell is rung first before presenting food to a dog for several days. The dog salivates
every time the bell rings alone.
 Which of the following is the most likely description of this paragraph?

A. premack’s principle
B. Operant conditioning
C. Classical conditioning
D. Reciprocal inhibition
E. Stimulus generalization
QUESTION 127

 There is a marked reduction in the total number of neurons especially in the


hippocampus.
 Which of the following is the most likely presenting impairment?

A. Mood
B. Memory
C. Motivation
D. Movement
E. Mannerisms
QUESTION 128

 Which of the following best defines intelligence?

A. Knowledge of a great many facts


B. The ability to think abstractly and learn from experience
C. The ability to get good grades in school
D. All the factors that make one person different from another
QUESTION 129

 Psychologists used the concept of motivation in several important ways


 Which of the following is the most likely example of these ways?

A. To split observable behaviour from internal states


B. To account for similarity in behaviour
C. To explain perseverance despite adversity
D. To explain reflexive responses
QUESTION 130

 The “three Cs”- challenge, commitment, and control – are the distinctive attitudes
that make up
A. The stage of resistance
B. Cognitive hardiness
C. Cognitive restructuring
D. An optimistic thinking
QUESTION 131

 Which of the following personality theories does emphasize unconsciousness


motivation?

A. Psychodynamic
B. Cognitive
C. Humanistic
D. Trait
E. Social
QUESTION 132

 Which of the following is the most effective technique in eliminating prejudice?

A. Education
B. Threat
C. Force
D. Legislation
E. Tax incentives
QUESTION 133

 Safa is on call and asked to evaluate a newly arrived patient in the emergency room. The patient is a 27 years old woman
who is accompanied by her worried parents and husband. They report she has not slept in 4 days, seems to be high on
something, and has been spending money recklessly. She called her elderly grandmother at 3 a.m. the previous morning,
greatly upsetting the 83 year old woman. The patient is an attorney working as an associate in a large law firm and has
been described as brilliant. Her recent behaviour is completely uncharacteristic, although she has always been energetic.
 Which of the following professions you prefer to call for consultation?

A. Neurologist
B. Psychiatrist
C. Psychologist
D. Social worker
E. Consultation liaison psychiatrist
QUESTION 134

 Nader is a solider experienced combat 2 years ago, he became overreacted to his daughter
unexpected surprises.
 Which of the following is the most likely explanation by which intensely traumatic experiences
can alter brain’s emotional responsiveness in detrimental ways?

A. Action potential
B. Neuronal plasticity
C. Memory encoding
D. Traumatic personality
E. Information processing
QUESTION 135

 Maged has recently ingested a recreational drug appears aggressive, paranoid, and restless. He said
that; I cannot sleep well as I feel my sleep very light and superficial. His wife said that, any thing can
awaken him.
 Which of the following sleep stages is the characteristic of Maged's sleep as measured in the sleep
lab?

A. NREM Stages 1 & 4


B. NREM Stages 1 & 2
C. NREM Stages 3 & 4
D. REM stage
E. NREM stages 1 & 3
QUESTION 136

 Abdou is 70 years old, lives alone after his wife death. Recently he accuses that, his
neighbour is deliberately whispering to avoid being heard. He beliefs that, his
neighbour talk about him in a bad manner.
 Which of the following is the most likely reason for Abdou’s paranoid state?

A. Most of us hate & burden by elderly people


B. Loneliness is a pathognomic factor in paranoia
C. Genetic tendency to paranoia increase by age
D. Diminished hearing is common ≥60 years old
QUESTION 137

 Joel heard the sound of a tree falling in the forest.


 Which of the following is the thing received by the Joel's brain?

A. Vibrations of the eardrums


B. Sound waves from the air
C. Neural activity in the sensory pathways
D. Sound waves travelling through the sensory pathways
QUESTION 138

 Ayman has tamper tantrum, which always seem to occur when his mother take him out
in public. His paediatrician offers a variety of tools that can help him. He asked his
mother not giving in to the temper tantrum and not letting him have what he wants
 Which of the following is the conditioning technique advised by the paediatrician?

A. Extinction
B. Reinforcement
C. Punishment
D. Recovery
QUESTION 139

 Recognition tests, which directly provide the test word, clearly by pass the search
and retrieval processes by which the Subject generates his
A. Recognition
B. Reconstruction
C. Recall
D. Relearning
E. Saving
QUESTION 140

 Farid tells you that he has found a way to improve his grades by stopping by his
professor’s office once a week to ask questions about the reading.
 Which of the following is the most likely type of intelligence Farid used?

A. Practical
B. Logic
C. Spatial
D. interpersonal
E. Emotional
QUESTION 141

 All during the day, Othman has repetitive thoughts about dirt and germs. To deal with these
repetitive images, he carefully washes and disinfects his hands precisely three times each hour.
 Which of the following basic emotions reflect Othman’s washing behaviour?

A. Fear
B. Anger
C. Disgust
D. Sadness
E. Surprise
QUESTION 142

 From your experience as a doctor, you learned that there is some people seem to “bounce back”,
even after severely traumatic experiences, while others are derailed by seemingly minor hassles.
 Which of the following could be the most likely reason for this difference in people’s response?

A. Type and numbers of stressors


B. Personal resources available
C. Consequences of the stressor
D. Cognitive appraisal of the stressor
E. Physical response tot the stressor
QUESTION 143

 Which of the following cortical regions is responsible for Your choice of major, your
plans for the summer, and your ability to answer test questions?

A. Thalamus
B. Hypothalamus
C. Occipital lobe
D. Medulla
E. Frontal lobe
QUESTION 144

 Nahla is 19 – year – old. She is at the first year of college. Since that time, her mother noticed
that, her attitude is changed. She start to adopt the behaviours, attitudes, & opinions of her
colleagues.
 Which of the following fields of psychology would be most likely to study the influence of
university on Nahlal?

A. Personality
B. cognitive
C. Social
D. Biological
E. Clinical
QUESTION 145

 Which of the following concepts raised by the model of artificial intelligence?

A. Our brain is a biological computer


B. Mind and brain are the same
C. Mind transplantation is an impossible dream
D. Personality is unchangeable
E. human’s brain is a simple primitive organ
QUESTION 146

 Which of the following statements is true regarding the mind’s functions?

A. There is a single centre for each of the major functions of the mind
B. Mind is composed of many specialized modules that work together
C. One brain network is specialized for every mental process
D. Least of the human cortex is devoted to integrating & interpreting senses
E. Neurons are inflexible has no ability to change or learn
QUESTION 147

 Which of the following symptoms suggest the presence of sleep disorder?

A. Snoring 2-3 days per week for 6 months


B. A REM period at the beginning of sleep
C. A brief cessation of breathing once or twice a night
D. Needing 9 hours of sleep each night in order to feel rested
E. Not remembering your dreams
QUESTION 148

 Which of the following is the best description of the new born mind?

A. Is a blank slate
B. Posses certain innate abilities
C. Develops slowly during the prenatal period
D. Can not develop new abilities
E. Resembles the adult brain but smaller
QUESTION 149

 Which of the following is the most important school of psychology which has
contributed a lot toward perception?

A. Psychoanalysis
B. Behavioural
C. Cognitive
D. Humanistic
E. Gestalt
QUESTION 150

 Which of the following regimens hospitals used to encourage desirable and healthy
patient behaviours e.g., grooming or taking medications?

A. Token economy
B. Money reinforcement
C. Early discharge plan
D. Discount program
QUESTION 151

 Which of the following memories is the result of instrumental /motor learning?

A. Semantic
B. Procedural
C. Episodic
D. Sensory
QUESTION 152

 Mohsen is a child 12 year old child. On testing his mental age was 15.
 Which of the following is the number of his IQ as Using the original IQ formula?

A. 125
B. 115
C. 100
D. 75
E. 98
QUESTION 153

 Which of the following is the correct example of the first component of emotion?

A. Increased heart rate, blushing & becoming pale


B. Fascial expressions, gestures and body language
C. Feeling of rage, sadness, happiness
D. Blaming someone, perceiving a threat
E. Smiling, crying, screaming for help
QUESTION 154

 Which of the following is the first stage of general adaptation syndrome?

A. Attention
B. Alarm
C. Alertness
D. Activity
E. Ambivalence
QUESTION 155

 Which of the following is the least detrimental factor of personality development?

A. Environment
B. Age
C. Learning
D. Life experiences
E. Culture
QUESTION 156

 Prejudice is a(n) -------, while discrimination is a(n) -------

A. Attitude / behaviour
B. Instinct / choice
C. Behaviour / emotion
D. Stimulus / response
QUESTION 157

 A 36 year-old-man complains of feeling miserable, unhappy and unmotivated about his life.
He believes that nothing he does for his employers or others changes the many adverse
stressors in his life.
 Which of the following is the most likely affected mind’s functions?

A. Cognition
B. Behaviour
C. Mood
D. Memory
E. Intelligence
QUESTION 158

 Which of the following is the site of release of hormone?

A. Synapse
B. Blood
C. Ventricles
D. Nucleus
E. Axon
QUESTION 159

 Many people cannot tolerate the closing of windows in the winter though they do not
know the “why of it”.
 Which of the following rationalize their behaviour?

A. Consciousness
B. Sub consciousness
C. Unconsciousness
D. Intelligence
E. Personality
QUESTION 160

 The mother of an 8-year-old girl brings her to the outpatient clinic as she is having increasing
difficulty in getting her daughter to go to school. The daughter had always been anxious about going
to school and that has gradually increased to such an extent she has not attended school for a month.
She is constantly worried that something bad will happen to her mother if she is away from her.
 Which of the following is the most likely attachment style controlling the girl’s school abstinence?

A. Ambivalent
B. Avoidant
C. Secure
D. Disordered
QUESTION 161

 A 45-year-old man who has had an emergency appendectomy informs you that he can see
tiny pink elephants dancing on the window-sill. He has a history of alcohol dependence
 Which of the following is the most likely impaired function?

A. Vision
B. Thinking
C. Perception
D. Mood
E. Behaviour
QUESTION 162

 A 10 – year – old boy likes to play football and neglects eating. His parents are advised
to allow him to play football only after eating
 Which of the following is the most likely learning concept used by parents?

A. Classical conditioning
B. Operant conditioning
C. Shaping
D. premack’s principle
E. Stimulus generalization
QUESTION 163

 A 80 – year – old widower living alone is brought to the accident and emergency department by his
neighbours. They have found him wandering at night on several occasions. They have also noticed
gradual deterioration in his mental functioning and self-care over the past 3 years. On examination,
he could recall his date of birth and address but is disoriented in time, place and person.
 Which of the following is the most likely affected type of memory?

A. Sensory
B. Long term
C. Working
D. Episodic
E. Declarative
QUESTION 164

 You: I am not good at my study


 Your friend: what do you need to be good?
 Which of the following is the differentiating point between your and your friend regarding
thinking?

A. You are less optimistic than him


B. He differentiate between thought and thinking
C. You are less intelligent than him
D. He is more organised than you
E. Both of you have moderate intelligence
QUESTION 165

 Patient uses hallucinogens drugs because he loves the feeling of fearful hallucinations
 Person likes to climb heights because he loves the feeling of hyperventilation
 Child rubs his genitalia against hard objects because he loves the feeling of pain
 Which of the following is the common factor between the three examples?

A. Risk taking
B. Low intelligence
C. Type A personality
D. Sensation seeking
E. Impulsivity
QUESTION 166

 Naya is a student in the final year of her secondary school. Her teachers has been giving her a
difficult time. She say that; “I am under stress” as though she was being squashed b a heavy
object.
 Which of the following is the most likely comment on Naya’s words?

A. Study materials are hard and you start late


B. Examination will finish soon, be patience
C. Teachers are harsh and hard every where
D. All students feel the same in their finals
E. Stress is not a situation but a response
QUESTION 167

 On a test, you often know the material well but find that the test is unfair or covers
material that the teacher did not indicate would be on the test?
 Which of the following statements describe your personality?

A. You have high external locus of control


B. You have high internal locus of control
C. You have low external locus of control
D. You have low internal locus of control
QUESTION 168

 A 60 year-old man attends the emergency room stating that he has been blind since the previous day. He
seems calm and indifferent. The blindness began suddenly after he saw his sons kicked each other. His
neurological exam is normal. The optic nerve and disc reveal no retinal problems. He refused any
psychological intervention. His son said that, we went to traditional healer because we believed that envy eye
is the main reason of his blindness.
 Which of the following is the main reason beyond the belief in traditional healers?

A. Sociocultural
B. Religious
C. Financial
D. Experience
E. Personality
QUESTION 169

 Which of the following is the most prevalent inhibitory neurotransmitter in the


CNS?

A. Acetyl choline
B. Serotonin
C. Glutamate
D. Epinephrine
E. GABA
QUESTION 170

 Which of the following is called the emotional brain?

A. Brain stem
B. Limbic system
C. Cerebellum
D. Cerebrum
E. Sympathetic system
QUESTION 171

 You wanted to sample the contents of pre-consciousness in a patient


 Which of the following is the most likely helpful?

A. Give him/her MRI scans


B. Do a priming experiment
C. Have him/her undergo psychoanalysis
D. Ask him/her to recall specific memories to consciousness
QUESTION 172

 You are a paediatrician and you want to encourage mothers to be aware of the
psychological raising up of their children.
 Which of the following is the most likely advice you will give to them?

A. Avoid mixing their children with lower class


B. Set strict rules for punishment
C. Stimulate the five senses of their neonates
D. Keep their neonates sleeping all the time
QUESTION 173

 On a dark night, we may hear the footsteps of a thief outside the house, where as in
reality, a cat may be passing by.
 Which of the following is the most likely description for what happened to us?

A. Hallucination
B. Illusion
C. Fantasy
D. Daydreams
E. Memory defect
QUESTION 174

 A 12 – year – old school girl is keen to learn cooking from her mother. As a result, she spends a
lot of time in the kitchen with her mother. A week ago, she burnt her fingers while trying to
fry chips. Since then, she avoids going near the cooker.
 Which of the following is the most likely happened with this girl?

A. Classical conditioning
B. Operant conditioning
C. Stimulus generalization
D. Stimulus discrimination
E. Reciprocal inhibition
QUESTION 175

 Which of the following is correctly arranged?

A. Encoding – Retrieval - Storage


B. Encoding – Storage – Retrieval
C. Storage – Encoding – Retrieval
D. Retrieval – Encoding – Storage
E. Storage – Retrieval – Encoding
QUESTION 176

 Which of the following statements is true about assimilation and accommodation?

A. Complimentary to each other


B. Opposite to each other
C. Equal to each other
D. Unrelated to each other
E. Reversible
QUESTION 177

 Which of the following is most likely statement describing the one’s ability to control
his/her emotional responses?

A. A personality trait that can not be changed


B. Largely a matter of hormones
C. Closely connected to intelligence
D. A skill that can be learned
QUESTION 178

 Which of the following statements is true about stress management?

A. It is learning about the connection between mind & body


B. It helps us control our life in a positive sense
C. It teaches us to avoid all kinds o stress
D. It gives us the security feeling
E. It need mainly internal locus of control
QUESTION 179

 Which of the following is the common factor shard by psychodynamic, humanistic &
cognitive theories of personality?

A. They all have a strong basis in psychological research


B. They all acknowledge the internal mental process underlying our personality
characteristics
C. They all view personality as largely unconsciousness
D. They all say that men & women have entirely different motives underlying their
behaviour
QUESTION 180

 We usually adapt our behaviour to the demands of the social situation. In addition, in
ambiguous situations we take our cues from the behaviour of others in that setting
 Which of the following is the most likely name for this?

A. Situationism
B. Dispositionism
C. Structuralism
D. Functionalism
E. Ritualism
QUESTION 181

 Omar is an intelligent 5 year old boy had been the only child in a family until a sister was
born. Following the birth, he began to wet his bed after being dry for 3 years. No physical
abnormality was detected.
 Which of the following professions you prefer to call for consultation?

A. Psychiatrist
B. Clinical psychologist
C. Social worker
D. Urologist
E. Baby sitter
QUESTION 182

 Millions more jolt their brain awake with the caffeine of their morning coffee, tea, or
cola and the nicotine is an accompanying cigarette.
 Which of the following is responsible for these substances' effect on the brain?

A. Lipids
B. Glucose
C. Neurotransmitters
D. Platelets
E. Cerebrospinal fluid
QUESTION 183

 Ahmed goes to the clinic and describes problems with voluntary motor function. He tells the doctor that he is
experiencing tremors and weakness that have led to a loss of function in his ankle. After running tests, the
doctor cannot find any medical reason for the symptoms which obviously cause the man significant distress and
impairment in his occupation (which requires him to stand all day). The individual is referred to a psychiatrist,
who shows that the symptoms lack internal consistency by having the individual perform an ankle plantar
flexion test both sitting (where he displayed weakness) and then asked the individual to stand on his toes.
 Which of the following is the most likely origin of Ahmed’s symptoms?

A. Unconsciousness
B. Consciousness
C. Pre consciousness
D. Sub consciousness
QUESTION 184

 Hassan is a neonatologist working in a paediatric hospital. He used to see premature


babies, babies born at risk, underweight babies.
 Which of the following will be the best advice you want to give to doctor Hassan in order
to promote babies’ healthy psychological development?

A. Sing to them
B. Touch them
C. Feed them on demand
D. Make eye contact with them
E. Let them sleep more
QUESTION 185

 Fathi is a football player suffered a sever injury during a competition with the
national team. He felt little pain until the end of the match. This is an example of
mind-body effect on pain.
 Which of the following is responsible for this effect?

A. Personality traits
B. Nature nurture interaction
C. Gate control theory of pain
D. Schemas of thinking
QUESTION 186

 Samir is a child go to his/her paediatric doctor’s office and get a particularly unpleasant injection,
during which the sight of the needle becomes associated with pain. As a result, the next time you need
a shot, he wince when he first see the needle.
 Which of the following is the most likely explanation for this persistent change in Samir’s behaviour?

A. Thoughts
B. Learning
C. Fear
D. Instincts
E. Reflex
QUESTION 187

 You are Samir's family doctor. You know him for several years ago. However, when you
meet him at the airport, he mistaken your name and called you with your brother's name.
 Which of the following is the most likely memory’s sin happened with Samir?

A. Blocking
B. Forgetting
C. Suggestion
D. Bias
E. Misattribution
QUESTION 188

 Which of the following advices is the most appropriate one to be given to a mother of
child with IQ 55 by a paediatrician?

A. Quite environment
B. Proper sensory stimulation
C. Psychosocial isolation
D. Early schooling
E. Excessive protein intake
QUESTION 189

 You fight with your father. You asked yourself what am I feeling right now? What my dad
feeling right now? What I do to control my emotion right now? How can I show respect to my
dad? How am I interpreting, evaluating & analysing what happened & why?
 Which of the following best what you do in this situation?

A. Analytical thinking
B. High morality
C. Flexible personality
D. Social intelligence
E. Emotional intelligence
QUESTION 190

 Suppose you are unprepared for class discussion, but the instructor calls on you. Your
heart races, your knees feel wobbly, and you feel the urge to run away.
 Which of the following best defined your body response?

A. Fight or flight response


B. Tend and befriend response
C. Cognitive hardiness
D. Phobic reaction
E. Generalized anxiety
QUESTION 191

 A 38 – year – old woman who was wandering on the street in the middle of the night is
brought to the accident and emergency department by her neighbours. She can not give any
details of her identity. However, her neighbours said that her husband has died 7 days ago.
 Which of the following is the most likely defensive mechanism used?

A. Denial
B. Projection
C. Regression
D. Dissociation
E. Acing out
QUESTION 192

 Ahmed is a family doctor refused to examine female patients


 Which of the following is descriptive of Ahmed’s behaviour?

A. Prejudice
B. Discrimination
C. Conformity
D. Obedience to authority
E. Dispositionism
QUESTION 193

 Which of the following attitudes we must develop about reports of amazing new
treatments, dramatic psychological breakthrough, and products that claim to help
you develop untapped potential?

A. Careless and lacking


B. Critical and sceptical
C. Supportive and encouraging
D. Refusing and stubborn
E. Avoiding and fearful
QUESTION 194

 Which of the following sets of factors is all associated with the perspective indicated?

A. Neuroscience, hormonal system, genetics; biological perspective


B. Life span, temperament, social learning; developmental perspective
C. Memory, personality, environment; behavioural perspective
D. perception, unconsciousness, self concepts; cognitive perspective
E. Mental health, mental disorder, mental imagery; trait perspective
QUESTION 195

 Which of the following concepts, the capacity of development of day dreaming


depends on?

A. Ego or I
B. Id
C. Superego
D. Libido
E. Complexes
QUESTION 196

 Which of the following needs is essential for proper psychological development?

A. Luxury life
B. Secure attachment
C. Infrequent touch
D. Motor stimulation
E. Early schooling
QUESTION 197

 Which one of the following happens to be an important factor in selectivity of


attention?

A. Boredom
B. Attitude
C. Concentration
D. Interest
QUESTION 198

 Which of the following stimuli is the most associable with illness according to
classical conditioning theory?

A. Sound
B. Light
C. Smell
D. Food
E. Colours
QUESTION 199

 Which of the following memories in which the materials are stored for later
retrieval?

A. Rote
B. Sensory
C. Working
D. Long term
E. Immediate
QUESTION 200

 Which of the following is the first step in problem solving?

A. Selecting a strategy
B. Avoiding pitfalls
C. Identifying the problem
D. Searching for analogies
E. Reading about the problem
QUESTION 201

 Which of the following best defines an individual who have a biological need for
higher levels of stimulation than do other people?

A. Impulsive & crazy


B. Emotional intelligent
C. Apathetic & cold
D. Risk taker
E. Sensation seeker
QUESTION 202

 Which of the following is the most likely response to threat in women in contrast to
men?

A. Confession and crying


B. Sympathetic and arousal
C. Fight and flight
D. Tend and befriend
E. Aggressive and facing
QUESTION 203

 Which of the following best defines “behaviour”, in most of the people?

A. Can be easily explained


B. Has multiple causes
C. Stems from unconsciousness desires
D. Depends on social influence
QUESTION 204

 Which of the following produced a decrease in conformity?

A. The majority was not unanimous in its judgement


B. The subject has to respond publically, rather than privately
C. The task was seen as difficult or ambiguous
D. The group was very large
QUESTION 205

 A 46-year-old woman told her doctor that she had read some undergraduate psychology
textbooks. She believed that her sudden and surprising outbursts of anger might be due to
her childhood experiences. She then talked at great length on the subject of child rearing
practices in different cultures.
 Which of the following perspectives of modern psychology this woman point to?
A. Developmental
B. Psychodynamic
C. Cognitive
D. Humanistic
E. Behavioural
QUESTION 206

 Depressed people as a rule have signs of decreased activity in the __________


hemisphere of their cerebral cortex. When they are given ECT to just one
hemisphere, it is given on the __________ side.

A. Left – Left
B. Left – Right
C. Right – Left
D. Right – Right
QUESTION 207

 A 40-year-old businessman states that over the past 2 years, he has trouble staying awake for
more than 2 hours before falling asleep. He has a hard time sleeping through the night.
Meanwhile, his performance at work is suffering.
 Which of the following is the most likely diagnosis?

A. Sleep apnoea
B. Narcolepsy
C. Insomnia
D. Hypersomnia
E. Depression
QUESTION 208

 A 5 year old boy is looking out of the train window. He appears perplexed and asks mother
why the tress near to him tend to move away fast whereas the trees far away tend to move
slowly.
 Which of the following is the most appropriate attitude the mother must take?

A. Order him to get away from the window


B. Explain the rule in quite voice & simple words
C. Hug him to limit his movement
D. Say to him, you are so young to understand
E. Close the window and stare to him
QUESTION 209

 We perceive a square as a square in whatever position it is held.


 Which of the following is the most likely responsible for this perceptual constancy?

A. Height
B. Size
C. Shape
D. Wight
E. Depth
QUESTION 210

 A 12 – year – old girl, who developed a phobia after being bitten by a non poisonous
snake, is exposed to rope repeatedly. Phobic symptoms gradually disappear.
 Which of the following is the most likely explanation for phobia improvement?

A. Generalization
B. Shaping
C. Extinction
D. Inhibition
QUESTION 211

 A 52-year-old housewife has a history of uncontrolled partial seizures. She is charged


by the police with shoplifting. At the police station, she states that he has no memory of
what happened. However, an eye witness saw her stealing something from a store. The
witness is required to match an image for her against a suspect in a line-up
 Which of the following functions will be used by the witness’s memory?

A. Recall
B. Recognition
C. Cuing
D. Guessing
E. Coding
QUESTION 212

 Patient saw a plasma and called it a blood


 Nurse said; no it is a plasma
 Which of the following is the right arrangement of components of thoughts?

A. Assimilation – Accommodation
B. Assimilation - Schema
C. Shema – Assimilation
D. Accommodation – Assimilation
E. Accommodation – Schema
QUESTION 213

 Amira is a diabetic child. She used to comply on her insulin for the sake of her own.
She does not need her mom to remind her.
 Which of the following is correct regard Amira’s compliance on treatment?

A. She is keen to be healthy


B. She is afraid from her mom
C. She is afraid from her doctor
D. She is intelligent obedient girl
E. She is young with good memory
QUESTION 214

 Marwan is a renal patient since 20 years ago. He used to make renal dialysis three
times per week. He is so exhausted both financially ad psychologically. Recently he
attend to the outpatient clinic complaining of headache and pain in the stomach.
 Which of the following is the most likely explanation for his recent symptoms?

A. General adaptation syndrome


B. Gate pain theory
C. Fight and flight response
D. Somatic depression
QUESTION 215

 On a test, you talk to your professors individually; ask them to give you some
pointers on what they consider to be especially important (& testable) in their
classes.
 Which of the following statements describe your personality?

A. You have high external locus of control


B. You have high internal locus of control
C. You have low external locus of control
D. You have low internal locus of control
QUESTION 216

 When a person joins a new group, such as a work group or a group of friends, there is always
an adjustment period during which the individual tries to discover how best to fit in.
 Which of the following is the main discovery you need for good adjustment to the group?

A. Social pressures
B. Social norms
C. Social background
D. Social class
E. Social motives
QUESTION 217

 Not all psychologists are therapists


 Which of the following fields of psychology concerned with dealing with patients?

A. Biological
B. Experimental
C. Teaching
D. Environmental
E. Engineering
QUESTION 218

 Which of the following is the site of release of neurotransmitter?

A. Synapse
B. Blood
C. Ventricles
D. Nucleus
E. Axon
QUESTION 219

 Which of the following is most consistent with a typical NREM dream?

A. You remember that all of the sounds and colours in your dreams were completely
pleasurable
B. You remember being chased by two dogs and cat
C. You remember that you had been dreaming about something, but the memory is not
vivid
D. You clearly recall walking along a mountain ridge and feared falling over a cliff
QUESTION 220

 Which of the following developments is belonged to Piaget?

A. Cognitive
B. Emotional
C. Motor
D. Kinaesthetic
E. Moral
QUESTION 221

 Which of the following statements is true about the “perceptual principle of


‘proximity”?

A. Stimuli coming from equal distance are perceived as part of the same figure
B. When stimuli are close together they tend to be grouped
C. The nearer an object is, the more likely it is to be perceived
D. The closer a stimulus is, the more likely it is to be attended
QUESTION 222

 A 10 year old boy helps his mother by washing the dishes after the family supper. His mother
rewards him by giving him an extra helping of the pudding. He quickly learns that every time
he helps his mother in any way he gets an extra treat and therefore starts doing that regularly.
 Which of the following is the most likely explanation for what happened to this boy?

A. Classical conditioning
B. Operant conditioning
C. Observational learning
D. Stimulus generalization
E. Avoidance learning
QUESTION 223

 Which of the following corresponds to; Learning, retention, recall and recognition?

A. Memory
B. Intuition
C. Intelligence
D. Thinking
E. Imagination
QUESTION 224

 Which of the following experiences, heuristic as a strategies or approaches to


problem solving based on?

A. Past
B. Future
C. Others
D. Present
E. Painful
QUESTION 225

 Which of the following is the most likely explanation for the fact that, some people
do better in competition than others?

A. Intelligence
B. Charisma
C. Body built
D. Motivation
E. Maturity
QUESTION 226

 According to the inverted U hypothesis, which of the following associated with


optimal performance?

A. High levels of stress


B. No stress
C. Moderate stress
D. Low stress
E. Specific stressors
QUESTION 227

 Which of the following is the most risky component of type A personality pattern of
behaviour?

A. Work holism
B. Perfectionism
C. Time urgency
D. Hostility
E. Insomnia
QUESTION 228

 Which of the following is the most likely definition of the term “culture”?

A. Antiquities of the past generations


B. Customs of the past generations
C. Complex blend of traditions shared by group of people
D. Religion shared by most population
E. One language shared by the whole population
QUESTION 229

 Fawzy is a 40 years old farm worker is hospitalized for an infectious pneumonitis. He seems to
be responding well to antibiotics but on the third day of his hospital stay he becomes restless,
irritable, and wants to leave. He appeared to be confused
 Which of the following professions would you want to consult for Fawsy’s case?

A. Psychologist
B. Social worker
C. Clinical psychologist
D. General psychiatrist
E. Consultation liaison psychiatry
QUESTION 230

 Some people seem to have high blood pressure because they have an anxiety response
while having their blood pressure taken at the doctor’s office.
 Which of the following parts of the nervous system produces this anxiety response?

A. Somatic
B. Sympathetic
C. Parasympathetic
D. Autonomic
E. Central
QUESTION 231

 Mahmoud is 47 years old diagnosed as sleep apnoea


 Which of the following is the most likely symptom of Ahmed?

A. Being alert during the day


B. Stop breathing periodically during the night
C. Tend to sleep very heavily during the night
D. Walk during his light sleep
QUESTION 232

 Madiha is a lady with a son diagnosed with down syndrome. A genetic counsel advices her to
shut him away in institution, where he can depend almost wholly on others to fulfil his basic
needs.
 Which of the following statements is the one against this advice?

A. Such institutions are expensive places


B. No cure for down syndrome has been found
C. Down syndrome are capable of considerable learning despite genetic impairment
D. No need for establish some personal independence for down cases
QUESTION 233

 Samar is a 55 year old lady complained of chronic back pain for more than 10 years.
She reported that her pain increase with all analgesics.
 Based on gate control theory of pain, which of the following interventions you want
to add to her management plan?
A. Improve the doctor patient relationship
B. Avoid gym training
C. Tell her that, you are illogic
D. Avoid natural dietary source of pain killers
QUESTION 234

 Lamia is an obese lady. Her psychiatric evaluation reveals that she must be admitted.
The dietician was not convince, as Lamia is complaint on her dietary regimen and has
not significant medical problems.
 Which of the following concepts will convince the dietician by Lamia’s admission?

A. Multiple cues conditioned with food


B. Relapsing course of obesity
C. Rejection sensitivity of obsess patients
D. Obesity is a mental disorder
QUESTION 235

 The patient rings the bell to the nurse, and once the nurse attend, he says; I am sorry, the
words are on the tip of my tongue, but I forget what I wanted from you.
 Which of the following is the most likely explanation for this TOT (tip of my tongue)
phenomena?

A. Both a recognition and recall fail


B. Improper match between retrieval & encoding
C. Uncommon in young age
D. Improper match between short & long term memory
E. Retrieval and encoding ae intact
QUESTION 236

 Shaima is a cancer patient. She is 45 years old. On her examination, she had
depressed mood with refusal of chemotherapy.
 Which of the following is her most expected mental schema?

A. Chemotherapy has negative impact


B. I cannot fight anymore
C. Nothing curable
D. I looked older after chemotherapy
QUESTION 237

 Mona and Adel were referred for marital therapy. Mona’s frequent jealous questioning of Adel about
his life outside of the marriage depressed and angered him. He used to beat her once he became angry.
The therapist advice Adel for anger management by practicing certain questions like; what kind of face
do I have when I am angry? What things do I say? Have I learnt anything about my anger ?
 Which of the following best defines these questions?

A. Emotional intelligence
B. Positive reinforcement
C. External Locus of control
D. Self realization
E. Cognitive interpretation
QUESTION 238

 Ramy is a man 50 years old. He lost his wife and children in a harsh car accident. He
presented to the emergency room freeze like a dead person, no movements, no words, and
with starring look. His vital data is within the normal expected range.
 Which of the following is the most likely explanation for Ramy’s case?

A. General adaption syndrome


B. Fight and flight response
C. Initial physiological arousal
D. General inhibition syndrome
E. Tend and befriend response
QUESTION 239

 Tarek fails in an interview tries to save his prestige by announcing that he did not
really want the job
 Which of the following defensive mechanisms identify Tarek’s behaviour?

A. Displacement
B. Denial
C. Rationalization
D. Reaction formation
E. Sublimation
QUESTION 240

 Being a college student, carries certain implicit assumptions about attending classes,
studying, and handing in papers before deadline.
 Which of the following best defines these assumptions?

A. Social pressure
B. Social identity
C. Social role
D. Social norms
E. Social situation
QUESTION 241

 Which of the following statements is the most likely to differentiate psychology from
other disciplines like psychiatry?

A. Psychologists must have doctoral degrees


B. Psychology focus on mental disorder
C. Psychologists do research
D. Psychology is a commercial not scientific discipline
E. Psychology is a broader field, covering all aspects of behaviour and mental processes
QUESTION 242

 Which of the following are the main three layers of the human brain?

A. survival, emotional, & thinking


B. frontal, parietal, & temporal
C. anterior, posterior, & lateral
D. left, right, & central
E. cerebral, cerebellar, & meningeal
QUESTION 243

 Which of the following characterize day dreams in contrast to fantasy?

A. Being conscious
B. Being unconscious
C. Being sub consciousness
D. Being less organized
E. Being wish fulfilment
QUESTION 244

 Jean Piaget used the term “schemata“ to refer to the cognitive structures underlying
organized patterns of

A. Concept
B. Temperament
C. Behaviour
D. Experience
E. Attachment
QUESTION 245

 Which of the following best defines perception?

A. psychological;
B. Physiological
C. Biological
D. Psychophysiological
E. Physical
QUESTION 246

 Which of the following behaviours correspond to operating a computer?

A. Learned
B. Instinctive
C. Impulsive
D. Compulsive
E. Primitive
QUESTION 247

 Which of the following best defines an extreme case of amnesia?

A. Regression
B. Rationalization
C. Displacement
D. Repression
E. Projection
QUESTION 248

 A math problem calls for finding the area of a triangle. You know the formula, so you
multiply the base times ½ the height.
 Which of the following techniques you used o solve this problem?

A. An algorithm
B. Heuristics
C. Trial & error
D. Deductive reasoning
E. Feedback
QUESTION 249

 Which of the following makes us act as we do?

A. Motivation
B. Intelligence
C. Thinking
D. Perception
E. Personality
QUESTION 250

 Which of the following is the most likely response to prolonged stress according to
general adaptation syndrome (GAS) theory?

A. Increased use of emotion-focused coping strategies


B. Gradual strengthening of the immune system
C. Exhaustion, illness, & possibly death
D. Enhanced mental & physical ability to combat the stressor
QUESTION 251

 Which of the following is the feeling of person with internal locus of control?

A. Location matters most when it comes to understanding behaviour


B. Internal feelings are more important than outward appearance
C. They are responsible for their own achievements
D. Emotional control is the best way of coping with stress
QUESTION 252

 Which of the following does the research say is most important in predicting initial
attraction?

A. Religious attitude
B. Personality
C. Physical attractiveness
D. Money
E. Nurturing qualities
QUESTION 253

 A 55-year-old-man, non executive director of an international company, attends the outpatient


psychiatric clinic for psychotherapy. During his interview he repeatedly tells his assessor “my
colleagues think I am a failure in my work and my personal life”.
 Which of the following perspectives of modern psychology must be adopted in his psychotherapy?

A. Cognitive
B. Behavioural
C. Psychodynamic
D. Humanistic
E. Trait
QUESTION 254

 Nehad has fallen down the stairs. She is unable to move the right side of her body
and she is struggling to talk.
 Which of the following is the most likely lesion Nehad's brain show us?

A. Right frontal lobe


B. Left frontal lobe
C. Right temporal lobe
D. Left temporal lobe
QUESTION 255

 Mira is playing with her sister. She asked her to close her eyes and imagine a giraffe.
She started to ask her about its details like; colour, tail, length, place, company etc.
 Which of the following techniques Mira used with her sister?

A. Priming
B. Mental rotation
C. Zooming in
D. Introspection
E. Imagination
QUESTION 256

 Eight – year – old Sarah wants to surprise mother by washing the lunch dishes. Mother
unintentionally corrects her. Sarah feel inadequate with low self confidence
 According to Erikson, Which of the following is main life challenge not fulfilled with Sarah’s
psychosocial development?

A. Trust
B. Competency
C. Intimacy
D. identity
E. Integrity
QUESTION 257

 Aly is a young swimmer does not like to dive into the pool, because the water feels
too cold.
 Which of the following is the best advice for Aly to do before diving into the pool so
the water will feel less cold?

A. Take a hot shower


B. Take a cold shower
C. Exercise vigorously
D. Meditate
QUESTION 258

 A 13 year old girl is bitten by a non poisonous snake while playing in the garden. Now
she is afraid of ropes and refuses to play in the garden.
 Which of the following is the most likely happened with this girl?

A. Extinction
B. Experimental neurosis
C. Stimulus discrimination
D. Stimulus generalization
E. premack’s principle
QUESTION 259

 Heidy is a 35 years old depressed woman. She complained of negative emotions with intrusive bad
automatic thoughts about herself. She can not move after her husband’s death. Every day she looks
at their wedding pictures and cry. You said to her you must move on and live your life. She said, “I
want but I cannot, could you give me a memory cleaner?"!
 Which of the following is the most likely memory's sin Heidy has?

A. Suggestion
B. Bias
C. Persistence
D. Transience
E. Misattribution
QUESTION 260

 Mohamed forgot the ruler and used the pen instead of the ruler to line
 Which of the following is the used problem solving strategy by Mohamed?

A. Algorithm
B. Heuristic
C. Schema
D. Creativity
QUESTION 261

 Adam is an athlete always want to be “Up” for a game competition – but how far up
should he be?
 Which of the following is the best advice for Adam in order to fulfil his aim?

A. Too little arousal improve performance


B. Too much arousal improve performance
C. Moderate arousal improve performance
D. No relation between arousal and performance
E. Intrinsic motivation is the key for better performance
QUESTION 262

 Dina is a conscientious student who never takes a lunch break, preferring to eat something while keeping ahead
on assigned reading. She says she is “dead set on getting a 4.0”. She carries her course texts in her backpack
though they are not required in class and is never without her cell phone. Hurrying to and from classes to be
sure she is never late, she feels anxious about delays and angry when traffic slows. She eats while checking her
email, reads assignments in bed, and often needs medication to help her relax and get some sleep.
 Which of the following is the most likely type of Dina's personality?

A. Type B
B. Type A
C. Type C
D. Obsessive
E. Anxious
QUESTION 263

 An 8-year-old boy previously dry starts wetting his bed. His parents separated
recently after the birth of his sister.
 Which of the following is the most likely used defensive mechanisms?

A. Denial
B. Projection
C. Dissociation
D. Displacement
E. Regression
QUESTION 264

 Which of the following make an individual a member of social group?

A. Learning
B. Maturation
C. Learning
D. Social class
E. Socialization
QUESTION 265

 Which of the following is the main idea we conclude from the “Necker Cube”
application in psychological research?

A. We have 3D brain
B. We have imagination
C. We have mind
D. We have intelligence
E. We have photo memory
QUESTION 266

 Which of the following are the main components of neuroscience?

A. Neurology, biology, histology & psychology


B. Psychology, psychiatry, immunology & neurology
C. Psychology, biology, physiology & microbiology
D. Neurology, biology, psychiatry & psychology
E. Biology, histology, neurology & psychology
QUESTION 267

 Which of the following is the hypothalamic part responsible for the regulation of the
circadian rhythm?

A. Ventromedial nucleus
B. Superchiasmatic nucleus
C. Anterior portion
D. Lateral portion
QUESTION 268

 Mother & father are sitting in the family room. Both observe eleven – year – old Mark
impulsively hit his younger brother on the ear. Neither parent scolds Mark or uses any other
form of mild correction for his aggressive behaviour.
 Which of the following is the most likely parenting style used by those parents?

A. Authoritative
B. Authoritarian
C. Permissive
D. Uninvolved
E. Overprotective
QUESTION 269

 Maya for a crazy reason, she want to paint her room yellow, so she get some samples
at the paint store. When she holds the sample against her white wall, it looks
different from the way it looks against the green curtain.
 Which of the following is the best attribution for this difference?

A. Perceptual constancy
B. Contrast effects
C. Sensory adaptation
D. Signal transduction
QUESTION 270

 A family bought a young cat recently. It initially respond to all members of the family and to
visitors. However, over time, the cat learnt to respond only to commands of two family members

 Which of the following is the most likely happened with this cat?

A. Stimulus generalization
B. Stimulus discrimination
C. Stimulus conditioning
D. Stimulus sensitization
E. Stimulus inhibition
QUESTION 271

 Recognizing is remembering something in its presence, whereas recalling is different.

 Which of the following best defines recalling?

A. Recognizing it in its presence


B. Remembering’ it in its absence
C. Reconstructing it in its presence
D. Relearning it in its absence
QUESTION 272

 Which of the following is the storage place for schemas “organized bodies of informa­
tion”?

A. Perceptual field
B. Cognitive field
C. Sensation
D. Memory
E. Emotional field
QUESTION 273

 Which of the following is the complement of emotions?

A. Thinking
B. Motivation
C. Behaviour
D. Memory
E. Perception
QUESTION 274

 Which of the following is the responsible for “fight & flight” response?

A. Hypothalamus
B. Hippocampus
C . Thalamus
D. Insula
QUESTION 275

 A 32 – year – old married woman becomes verbally and physically aggressive towards her 5 –
year-old child. Her husband is alcoholic. He observed this whenever he came back from the
pub late in the evening
 Which of the following is the most used defensive mechanisms by this wife?

A. Denial
B. Displacement
C. Regression
D. Acting out
E. Dissociation
QUESTION 276

 Which of the following is not an element of the social context?

A. People
B. Objective reality
C. Interaction between people
D. Behavioural setting
QUESTION 277

 After discharge from hospital a 75– year-old widow become progressively more suspicious over
a 3 month period. She telephones her son, making accusations a number of times every day. She
refuses to allow her homecare worker into her house. She is also sleeping poorly at night.
 Which of the following is the most likely affected function of the mind?

A. Mood
B. Behaviour
C. Consciousness
D. Thinking
E. Memory
QUESTION 278

 A 35 years old male has recently experienced a severe headache accompanied by a


perception of strange smells.
 Which of the following is the most likely affected brain lobe?

A. Frontal
B. Parietal
C. Occipital
D. Temporal
QUESTION 279

 Tahany is a student in her first year of college of medicine. She came to outpatient clinic complaining of
anxiety and fear. She is unable to master her study subjects. She said; my comprehension is weak and I
forgets quickly.
 Which of the following is the most helpful advice for her in order to improve concentration and memory?

A. Study only in weekends


B. Sleep at the same time every day
C. Use coffee & tea as much as possible
D. Avoid study and discussion with colleagues
E. Make connections between old & new information
QUESTION 280

 Very often, children think that everything around them has a life similar to theirs.
Tree, stones, and doors all have lives for them.
 Which of the following is the correct name of this pattern of thinking?

A. Egocentrism
B. Personification
C. Animistic thinking
D. Accommodation
E. Assimilation
QUESTION 281

 During a crime scene, a detective finds a sip of paper with three symbols printed on it in
ink. She cannot identify the source of the figures or which orientation is up. Thus she
cannot determine if the figures are the numbers 771 or the letters ILL. She has to guess at
the meaning of the figures.
 Which of the following is the most likely helpful type of perception?

A. Data driven
B. Bottom – up
C. Top – down
D. Stimulus driven
QUESTION 282

 you ere going to use observational learning concept in developing a program to


prevent violence among middle school children
 Which of the following strategy you will might use to fulfil your aim?

A. Have children role paly non aggressive solutions in interpersonal problems


B. Punish children for aggressive acts performed at school
C. Have a children punch a Bobo doll, to get the aggression out of their system
D. Have children watch videos of aggressive children who are not being reinforced for
their aggressive behaviour
QUESTION 283

 A person, who has had a head injury, is unable to recall the events that took place
before the accident.
 Which of the following is the most likely diagnosis?

A. Trauma
B. Phobia
C. Retrograde amnesia
D. Aphonia
E. Astasia Abasia
QUESTION 284

 Manal had night shift at the lab, the technician told her that, there is a wife refused blood transfusion.
Her husband was operating on and she is the only way to save his life. She said that, I love him and I
want to help but the problem is in my health. I am so fragile & easily fatigued and if I give him blood
I will not withstand to serve him postoperatively. Which is the problem solving obstacle
 Which of the following is the major problem solving obstacle with this wife?

A. Self imposed limitations


B. Confirmation bias
C. Blood phobia
D. Antisocial personality
E. Lowe intelligence
QUESTION 285

 You see death ECG images at the monitor of your patient in the intensive care
 Which of the following is true regarding your emotional response?

A. Fast unconsciousness routes visual information from thalamus to amygdale


B. Fast consciousness routs visual information through amygdale to thalamus
C. Slow unconsciousness pathway involve visual cortex
D. Slow consciousness pathway from cortex to thalamus
QUESTION 286

 Fahd is a student usually stops for conversations or lunch with friends. He doesn't lug books
to campus unless he needs them, dose not drive in a hurry, and arrives at class on time if not
early most of the time. Though not a perfectionistic, Fahd somehow earns grades as good.
 Which of the following is Fahd’s personality type?

A. Type A
B. Type B
C. Type C
D. Dependent
E. Antisocial
QUESTION 287

 Ayman is a high school student wanted to join the college after passing his annual examination. But
to his utter disappointment, he fails the examination. When he was asked that why he has not joined
the college, he cautioned his disappointment by stating that college is a waste of time and money.
 Which of the following is the most likely dense mechanism used by Ayman ?

A. Regression
B. Identification
C. Projection
D. Sublimation
E. Rationalization
QUESTION 288

 Suppose that you find yourself in an interview, with the possibility of being hired for the job of
your dreams. Afterward, the interviewer suggests that you go to such together in the hospital
Cafeteria. Will you order a sandwich, a salad, or a full course meal?
 Which of the following is the primarily determinant of your behaviour?

A. social reality
B. Social role
C. Social norms
D. Social situation
E. Social pressure
QUESTION 289

 A 35-year-old married man was admitted following an attendance in the accident and emergency
department after a fall. He informed the nursing staff that he had been drinking heavily for the past
5 years. He was not happy with his life and wants to stop his drinking altogether. The physical
investigations reveal severe hepatic and renal impairments.
 Which of the following professions you want to consult for this patient?

A. General psychiatrist
B. Clinical psychologist
C. Consultation liaison psychiatrist
D. Neurologist
E. Social worker
QUESTION 290

 Which of the following is the concerned part of the nervous system by which you
picks up dog’s barking voice and its teeth image and delivers both to your brain?

A. PNS
B. Spinal cord
C. Thalamus
D. Occipital cortex
E. Brain stem
QUESTION 291

 Which of the following is most clearly under the control of a circadian rhythm in
most organisms?

A. Sleep
B. Storage of body aft
C. Migration
D. Mating
E. Pain
QUESTION 292

 A 8-year-old boy needs to be coaxed to go to school, and often, while there, he complains of
severe headaches or stomach pain. Sometimes his mother has to take him home because of
his symptoms. At night he tries to sleep with his parents. When they insist he sleep in his
own room, he says there are monsters in his closet.
 Which of the following is the most likely problem in this child?
A. Attachment
B. Thinking
C. Intelligence
D. Mood
E. Behaviour
QUESTION 293

 An object that has been constituted percep­tually as a permanent and stable thing continue to
be perceived as such, regardless of illumi­nation, position, distance etc.
 Which of the following best defines this kind of stability of the environment experienced by
human beings?

A. Depth constancy
B. Size constancy
C. Perceptual constancy
D. Shape constancy
E. Colour constancy
QUESTION 294

 Which of the following behaviours correspond to nursing a new born?

A. Learned
B. Instinctive
C. Impulsive
D. Compulsive
E. Primitive
QUESTION 295

 Memory and remembering denote the same comprehensive process.


 Which of the following is the main components for both?

A. Learning, Retaining, Recall and Recognition


B. learning, imagination, recall and problem solving
C. Thinking, Intuition, Intelligence and Learning
D. Imagination, Intuition, Learning and Retention
E. Intuition, Imagination, thinking and Problem Solving
QUESTION 296

 Which of the following you can use to calculate your grades point average?

A. Algorithms
B. Heuristic
C. Memos
D. Bias
QUESTION 297

 Which of the following is the true statement regarding motivation?

A. It relates culture to behaviour


B. It accounts for variability in behaviour
C. It connects observable behaviour to environment
D. It is mostly internally more than externally
E. It is mostly biologically than learned
QUESTION 298

 Which of the following is the most likely feature of resilient people?

A. Are healthy & adaptive despite problems


B. Have learned to control other people
C. Do nearly perfect work
D. Give up & withdraw in the face of failure
QUESTION 299

 A 28-year-old woman presents with a 2 week history of being unable to speak, which began
suddenly after witnessing an assault. On examination, she is able to mouth words, but is
unable to vocalize. She is noted to cough audibly.
 Which of the following is the most likely defensive mechanisms used by this woman?

A. Regression
B. Acting out
C. Displacement
D. Projection
E. Dissociation
QUESTION 300

 Which of the following is the correct statement regarding social psychology?

A. It concerns with how individuals affect each other


B. Try to understand behaviour within its family context
C. It constructs the objective social reality
D. It highlights the intrapersonal factors in explain people’s behaviour

You might also like